Déterminant

Calculs

($\star^{1/2}$) Correction

Montrer que le déterminant d'une matrice antisymétrique d'ordre impair est nul.

Correction

Soit $A\in \MM_{2p+1}(\K)$ telle que $A$ est antisymétrique, alors $$\det(A)=\det({\,}^t A)=\det(-A)=(-1)^{2p+1}\det(A)=-\det(A)\Longrightarrow\boxed{\det(A)=0\,}.$$

($\star$) Correction

Soient $A \in \MM_{2n} (\mathbb{R})$ antisymétrique et $J \in \MM_{2n} (\mathbb{R})$ la matrice dont tous les coefficients sont égaux à 1. Montrer que: $$\forall x \in \mathbb{R},\quad\det (A + xJ) = \det A.$$

Correction

Notons $A=(C_1\,C_2\cdots C_{2n})$ avec $C_j\in \MM_{2n\,1}(\R)$ la $j$ième colonne de la matrice $A$, notons aussi $Z=\begin{pmatrix} 1\\\vdots\\1 \end{pmatrix}\in \MM_{2n\,1}(\R)$ de sorte que $J=(Z\,\cdots Z)$ on a: $$\begin{array}{lcl} \forall x\in \R,\quad\,\det(A+xJ)&=&\det( C_1+xZ,\cdots,C_{2n}+xZ)=\det(C_1+xZ,C_2-C_1,\cdots,C_{2n}-C_1)\\ &=&\det(C_1,C_2-C_1,\cdots,C_{2n}-C_1)+x\det(Z,C_2-C_1,\cdots,C_{2n}-C_1)\\ &=&\det(A) +x \det(Z,C_2-C_1,\cdots,C_{2n}-C_1) \end{array} $$ Donc $f(x)=\det (A+xJ)$ est une fonction affine en $x$, i.e. $f(x)=\det(A)+ \alpha x$.
d'autre part, On a: $$\forall x\in \R,\quad \det (A+xJ)=\det ({\,}^t (A+xJ))=\det (-A+xJ)=(-1)^{2n}\det(A-xJ)=\det(A-xJ)$$ donc $$\forall x\in \R,\quad f(-x)=\det(A)-\alpha x=f(x)\Longrightarrow \alpha =0.$$

($\star$) Correction

Soit $A \in \MM_n (\mathbb{K})$ de colonnes $C_1 , \ldots ,C_n $. Calculer le déterminant de la matrice $B$ de colonnes $$C_1 - C_2 ,C_2-C_3, \ldots ,C_{n - 1} - C_n ,C_n - C_1 $$

Correction

Il suffit de remarquer que la somme de colonnes de $B$ vaut $0$.

($\star$) Correction

Calculer les déterminants suivante (on les mettra sous forme factorise si possible): $$\mathbf{a)\,}\, \begin{vmatrix} a+b & b+c & c+a \\ a^2+b^2 & b^2+c^2 & c^2 + a^2 \\ a^3+b^3 & b^3+c^3 & c^3+a^3 \end{vmatrix}, \qquad \mathbf{b\,},\begin{vmatrix} xy & x^2 & y^2 \\ x^2 & y^2 & xy \\ y^2 & xy & x^2 \end{vmatrix}.$$

Correction

$\mathbf{a)\,}\,$ On fait $C_2\leftarrow C_2-C_1,\,C_3\leftarrow C_3-C_1$, on obtient, $$ \begin{array}{ll} D&=(c-a)(c-b)\begin{vmatrix} a+b & 1& 1 \\ a^2+b^2 & c+a & c+b \\ a^3+b^3 & c^2+ac+a^2 & c^2+cb+b^2 \end{vmatrix}\\ &\\ &=(c-a)(c-b)(b-a)\begin{vmatrix} a+b & 1& 0 \\ a^2+b^2 & c+a & 1 \\ a^3+b^3 & c^2+ac+a^2 & a+b+c \end{vmatrix}\\ &\\ &=(c-a)(c-b)(b-a)\left[ (a+b)\big((c+a)(a+b+c)-c^2-ac-a^2\big)-\big( (a^2+b^2)(a+b+c)-a^3-b^3\big)\right]\\ &\\ &=\boxed{\,2abc(c-a)(c-b)(b-a)\,} \end{array}$$ $\mathbf{b)\,}\,$ On développe selon la 1ère ligne $$ D= xy(y^2x^2-x^2y^2)-x^2(x^4-xy^3)+y^2(x^3y-y^4)=-x^3(x^3-y^3)+y^3(x^3-y^3)=\boxed{\,-(x^3-y^3)^2\,}. $$

($\star$) Correction

Calculer les déterminants suivante (on les mettra sous forme factorise si possible): $$ \mathbf{a)\,}\, \begin{vmatrix} 1& 1 & 1&1 \\ 1 &1 &\cos(2c)&\cos(2b) \\ 1 & \cos(2c) & 1&\cos(2a)\\ 1&\cos(2b)&\cos(2a)&1\end{vmatrix}, \mathbf{b)\,} \quad\quad \left| \begin{array}{cccc} 0 & 1 & 1 & 1 \\ 1 & 0 & a^2 & b^2 \\1 & a^2 & 0 & c^2 \\ 1 & b^2 & c^2 & 0 \end{array} \right|.$$

Correction

$\mathbf{a)\,}\,$ On soustrait la 1ère ligne des lignes suivantes, puis on développe selon la 1ère colonne $$D=\begin{vmatrix} 0 &\cos(2c)-1&\cos(2b)-1 \\ \cos(2c)-1 & 0&\cos(2a)-1\\ \cos(2b)-1&\cos(2a)-1&0\end{vmatrix}= \begin{vmatrix} 0 &-2\sin^2(c)&-2\sin^2(b) \\ -2\sin^2(c) & 0&-2\sin^2(a)\\ -2\sin^2(b)&-2\sin^2(a)&0\end{vmatrix}=\boxed{-16\sin^2(a)\sin^2(b)\sin^2(c)}$$ $\mathbf{b)\,}\,$ $$ \left| \begin{array}{cccc} 0 & 1 & 1 & 1 \\ 1 & 0 & a^2 & b^2 \\0 & a^2 & -a^2 & c^2-b^2 \\ 0 & b^2 & c^2-a^2 & -b^2 \end{array} \right|= \left| \begin{array}{ccc} 1 & 1 & 1 \\ a^2 & -a^2 & c^2-b^2 \\ b^2 & c^2-a^2 & -b^2 \end{array} \right|= \left| \begin{array}{ccc} 1 & 0 & 0 \\ a^2 & -2a^2 & c^2-b^2-a^2 \\ b^2 & c^2-a^2-b^2 & -2b^2 \end{array} \right| $$ ce qui donne $$D= 4a^2b^2-(c^2-b^2-a^2)^2=\big( (a+b)^2-c^2\big)\big(c^2-(a-b)^2\big)=\boxed{(a+b+c)(a+b-c)(c+a-b)(c-a+b)}$$

($\star\star\star$) Correction

Soit $P=\dsum_{k=0}^np_kX^k\in \R_n[X]$. Soient $(a_i)_{i\in \inter{0,n}},\,(b_j)_{j\in \inter{0,n}}$ deux familles de réels.
On considère la matrice $M=(m_{i\,j})_{(i,j)\in \inter{0,n}^2}\in \MM_{n+1}(\R)$, avec $m_{i\,j}=P(a_i+b_j)$. Calculer $\det (M)$.

Correction

On va essayer d'écrire la matrice $M$ comme le produit de plusieurs matrices dont les déterminants sont plus faciles à calculer. $$\forall (i,j)\in \inter{0,n}^2,\quad m_{i\,j}=P(a_i+b_j)=\dsum_{k=0}^n\underset{c_{k\,j}}{\underbrace{\dfrac{b_j^k}{k!}}} \underset{d_{i\,k}}{\underbrace{P^{(k)}(a_i)}}\Longrightarrow M=D\times C $$ avec $$D=\left(d_{i\,j}=P^{(j)}(a_i)\right)_{0\leq i,j\leq n},\,\,C=\left(c_{i\,j}=\dfrac{b_j^i}{i!}\right)_{0\leq i,j\leq n}$$ On commence à nouveau avec la matrice $D$, $$\forall (i,j)\in \inter{0,n}^2,\quad d_{i\,j}=P^{(j)}(a_i)=\dsum_{k=0}^n\underset{a_{i\,k}}{\underbrace{\dfrac{a_i^k}{k!}}}\underset{b_{k\,j}}{\underbrace{P^{(j+k)}(0)}}\Longrightarrow D=A\times B $$ avec $$A=\left(a_{i\,j}=\dfrac{a_i^j}{j!}\right)_{0\leq i,j\leq n},\,\,B=\left(b_{i\,j}=P^{(i+j)}(0)\right)_{0\leq i,j\leq n}$$ On en déduit $M=ABC$ ce qui donne, $\det(M)=\det(A)\det(B)\det(C)$.

  • Calcul de déterminant de $A$, $$ \begin{array}{lcl} \det(A)&=&\dsp \begin{vmatrix} \dfrac{a_0^0}{0!}&\dfrac{a_0^1}{1!}&\cdots&\cdots&\dfrac{a_0^n}{n!}\\ \dfrac{a_1^0}{0!}&\dfrac{a_1^1}{1!}&\cdots&\cdots&\dfrac{a_1^n}{n!}\\ \vdots& \vdots&\vdots&\vdots&\vdots\\ \dfrac{a_n^0}{0!}&\dfrac{a_n^1}{1!}&\cdots&\cdots&\dfrac{a_n^n}{n!} \end{vmatrix}\\ &&\\ &=&\dsp\dfrac{1}{1\times 1!\times 2!\cdots \times n!} \begin{vmatrix} 1&a_0&\cdots&\cdots&a_0^n\\ 1&a_1&\cdots&\cdots&a_1^n\\ \vdots& \vdots&\vdots&\vdots&\vdots\\ 1&a_n&\cdots&\cdots&a_n^n \end{vmatrix}=\dfrac{V_{n+1}(a_0,\cdots,a_n)}{1\times 2\times 6\cdots\times n!} \end{array}$$
  • Calcul de déterminant de $B$, puisque $P$ est de degré inférieure ou égale à $n$ alors $P^{(k)}(0)=0$ dès que $k>n$ et $P^{(n)}(0)=n!p_n$. $$ \begin{array}{lcl} \det(B)&=& \begin{vmatrix} P^{(0)}(0)&P^{(1)}(0)& &\cdots &\cdots &P^{(n)}(0)\\ P^{(1)}(0)&P^{(2)}(0)& &\cdots &P^{(n)}(0)&P^{(n+1)}(0)\\ \vdots & & & \diagup& &\\ \vdots & & \diagup & & \diagup &\\ \vdots & \diagup & & \diagup &&\\ \vdots & &\diagup &&&\\ P^{(n)}(0)&P^{(n+1)}(0)& & &\cdots&P^{(2n)}(0) \end{vmatrix}\\ &&\\ &=&\begin{vmatrix} P^{(0)}(0)&&\cdots&\cdots&\cdots &n!p_n\\ P^{(1)}(0)&&\cdots& &n!p_n&0\\ \vdots & & & \diagup& \diagup &\\ \vdots & &\diagup& \diagup &&\\ \vdots & \diagup & \diagup &&&\\ n!p_n&0&&\cdots&\cdots&0 \end{vmatrix}=(n!\,p_n)^{n+1}(-1)^{\frac{n(n+1)}{2}} \end{array}$$
  • Pour le déterminant de $C$, le calcul est identique à celui de $A$. $$\det(C)=\dfrac{V_{n+1}(b_0,\cdots,b_n)}{0!\times 1!\cdots\times n!}= \prod_{k=0}^n\dfrac{1}{k!}\prod_{0\leq i < j\leq n}(b_j-b_i)$$

On en déduit, $$\boxed{\det(M)=(-1)^{\frac{n(n+1)}{2}}\prod_{k=0}^n\dfrac{n!\,p_n}{k!^2}\prod_{0\leq i< j\leq n}(a_j-a_i)(b_j-b_i)}.$$

($\star$) Correction

Soit $A(x)=(a_{i\,j})\in \MM_n(\K)$ telle que $$\forall (i,j)\in \inter{1,n}^2,\quad a_{i\,i}=1+x^2,\quad a_{i\,j}=x \text{ si }\abs{i-j}=1,\quad a_{i\,j}=0\text{ dans les autres cas}.$$ Calculer $\det (A)$.

Correction

On note $D_n$ le déterminant de la matrice $A_n(x)$ (de taille $n$). Soit $n\in \N^*$, on a $$D_{n+2}=\begin{vmatrix} 1+x^2&x&0&\cdots&0\\ x& 1+x^2&x&\ddots&\vdots\\ 0&\ddots&\ddots&\ddots&0\\ \vdots& \ddots& &\ddots &x\\ 0&\cdots& 0&x&1+x^2 \end{vmatrix} $$ Puis en développant selon la 1ère colonne, on trouve $$D_{n+2}=(1+x^2)D_{n+1}-x^2D_n$$ donc si $x\not\in \{-1,1\}$, $D_n =a x^{2n} +b$, après calcul: $$\forall n\geq 2,\, D_n = \dfrac{x^2}{x^2-1}x^{2n}-\dfrac{1}{x^2-1}.$$ sinon ($x\in \{-1,1\}$) $D_n =an+b=n+1$

($\star$) Correction

Calculer le déterminant d'ordre $n$ : $\Delta(x)=\left| \begin{array} {cccc} a_1+x & a_1 & \ldots & a_1 \\ a_2 & a_2+x & \ldots & a_2\\ \vdots & \vdots & \ddots & \vdots \\ a_n & a_n & \ldots & a_n + x \end{array} \right|$

Correction

On soustrait la 1ère colonne des colonnes suivantes, ce qui donne, $$ \Delta(x)=\left| \begin{array} {cccc} a_1+x & -x & \ldots & -x \\ a_2 & & & \\ \vdots & & xI_{n-1} & \\ a_n & & & \end{array} \right|=\left| \begin{array} {cccc} a_1+a_2+\cdots+a_n+x & 0 & \ldots & 0 \\ a_2 & & & \\ \vdots & & xI_{n-1} & \\ a_n & & & \end{array} \right|=\boxed{x^n+\left(\dsum_{i=1}^na_i\right)x^{n-1}\,}. $$

(Déterminant de Vandermonde) Correction

Soient $n\in \N^*$ et $x_1,\cdots,x_n\in \K$. Calculer le déterminant suivant: $\Delta_n(x_1,\cdots,x_n)=\begin{vmatrix} 1 & x_1 & \cdots & x_1^{n-1} \\ 1 & x_2 & \cdots & x_2^{n-1} \\ \vdots & \vdots & \ddots & \vdots\,\,\, \\ 1 & x_n & \cdots & x_n^{n-1} \end{vmatrix}. $

Correction

On effectue les opérations suivantes $C_j\longleftarrow C_j-x_nC_{j-1}$ pour $j\in \inter{2,n}$, ce qui donne $$ \Delta(x_1,\cdots,x_n)=\begin{vmatrix} 1 & x_1-x_n & \cdots & x_1^{n-2}(x_1-x_n) \\ 1 & x_2-x_n& \cdots & x_2^{n-2}(x_2-x_n) \\ \vdots & \vdots & \ddots & \vdots\,\,\, \\ 1&x_{n-1}-x_n&\cdots& x_{n-1}^{n-2}(x_{n-1}-x_{n})\\ 1 & 0 & \cdots & 0 \end{vmatrix}=\prod_{i=1}^{n-1}(x_i-x_n)\Delta_{n-1}(x_1,\cdots,x_{n-1}). $$ Ensuite par récurrence sur $n$, on montre $$\boxed{\Delta(x_1,\cdots,x_n)=\prod_{1\leq i< j\leq n}(x_j-x_i)\,}.$$

($\star\star$) Correction

Soit $n\in \N^*$ et $(a_1,\cdots,a_n)\in \R^n$ tels que: $a_1< a_2\cdots< a_n$. On pose pour $x\in \R,~~$ $$P(x)=\begin{vmatrix}x & a_2 & a_3 & \dots & a_n \\ a_1 & x & a_3&\dots & a_n \\ \vdots& a_2 & \ddots& & \vdots\\ \vdots&\vdots& & \ddots& a_n \\ a_1 & a_2 & a_3\dots & a_{n-1} & x \end{vmatrix}$$ Calculer $P(a_k)$ ($1\leq k\leq n$). En déduire que $P$ admet $n$ racines réelles distincts.

Correction

Soit $k\in \inter{1,n}$, pour calculer $P(a_k)$ on soustrait la 1ère ligne des lignes suivante, ce qui donne: $$ P(a_k)=\begin{vmatrix} a_k &a_2 &\cdots & a_{k-1}&a_k &a_{k+1}&\cdots &a_n\\ a_1-a_k &a_k-a_2&0 & \cdots & \cdots &\cdots & \cdots &0\\ \vdots & 0 &\ddots & \ddots & & (0) & &\vdots \\ & \vdots& \ddots & a_{k}-a_{k-1} & \ddots & & &\vdots \\ a_1-a_k & \vdots & & \ddots &0 & \ddots & &\vdots\\ \vdots & \vdots& & (0) & \ddots & a_{k}-a_{k+1} & \ddots & \vdots \\ & \vdots & & & & \ddots & \ddots &0\\ a_1-a_k & 0 & \cdots & \cdots & \cdots & \cdots & 0 &a_k-a_n \end{vmatrix}$$ Puis on développe selon la 1ère colonne, en remarquant que chaque cofacteur d'indice $(i,j)$ est nul sauf si $j=k$ car il contient une ligne nulle (la $k$ère ligne de déterminant $P(a_k)$), on obtient alors, $$P(a_k)=(-1)^{k+1}(a_1-a_k)\begin{vmatrix} a_2 &a_3&\cdots & a_{k-1}&a_k &a_{k+1}&\cdots &a_n\\[2mm] a_k-a_2&0 &\cdots& \cdots & \cdots &\cdots & \cdots &0\\ 0 &a_k-a_3 & 0 & \cdots& & (0) & &\vdots \\\\[2mm] 0 & \ddots& \ddots & & & & &\vdots \\ \vdots& &\ddots & a_{k}-a_{k-1} & 0& \ddots & &\vdots \\ \vdots& & (0) & & \ddots & a_{k}-a_{k+1} & \ddots & \vdots \\ \vdots & & & & & \ddots & \ddots &0\\ 0 & \cdots & \cdots & \cdots & & \cdots & 0 &a_k-a_n \end{vmatrix}$$ Ensuite on développe selon la $(k-1)$ère colonne, $$P(a_k)=(-1)^{k+1}(-1)^k(a_1-a_k)a_k \begin{vmatrix} a_k-a_2&0 & \cdots &0\\ 0 &\ddots & 0 &\vdots\\ \vdots & \ddots & \ddots & 0\\ 0 & & 0 &a_k-a_n \end{vmatrix}=\boxed{ \,a_k\prod_{j\neq k}(a_k-a_j)\,}$$ La fonction $x\in \R\longmapsto P(x)$ est une fonction polynomial en $x$ unitaire de degré $n$ (par récurrence sur $n$) donc continue. On remarque que le signe de $P(a_k)=(-1)^{n-k}$ puisque $0< a_1\cdots< a_n$. Donc en utilisant le théorème des valeurs intermédiaire, on trouve $$\forall k\in \inter{1,n-1},~~\exists \theta_k\in ]a_k,a_{k+1}[,\text{ tel que } P(\theta_k)=0,$$ soit $n-1$ racine distincts de $P$. Il faut maintenant cherche le $n$ère racine!
Un calcul simple nous donne $P(0)=\dsp (n+(-1)^n)\prod_{i=1}^na_i\geq 0$ ce qui donne :

  1. Si $n$ est paire alors $P(0)P(a_1)< 0\Longrightarrow \exists \theta_n\in ]0,a_1[$ tel que $P(\theta_n)=0$.
  2. Si $n$ est impaire alors $\limiteX{x}{-\infty}P(x)=-\infty$ et $P(0)>0$ donc $\exists \theta_n\in ]-\infty,0[$ tel que $P(\theta_n)=0$.

Ce qui donne $P$ admet exactement $n$ racine réelles distincts.

(Mines Ponts 2016) Correction

Soient $A \in \MM_n (\mathbb{R})$ ($n \geqslant 2$) de colonnes $A_1 , \ldots ,A_n $ et $B \in \MM_n (\mathbb{R})$ de colonnes $B_1 , \ldots ,B_n $ déterminées par $B_j = \sum\limits_{i \ne j} {A_i } $.
Exprimer $\det B$ en fonction de $\det A$.

Correction

Soit $\BB$ la base canonique de $\MM_{n\,1}(\R)$, alors on a: $$\det B=\det_\BB(B_1,B_2,\cdots,B_n)=\det_\BB\left(\sum_{i=1}^nB_i,B_2,\cdots,B_n\right),$$ or $\dsum_{i=1}^nB_i=(n-1)\dsum_{i=1}^nA_i$. Donc $$\begin{array}{lcl} \det(B)&=&\dsp (n-1)\det_\BB\left(\sum_{i=1}^nA_i,B_2,\cdots,B_n\right)\\ &&\\ &=&\dsp (n-1)\det_\BB\left(\sum_{i=1}^nA_i,-A_2,\cdots,- A_n\right)\\ &&\\ &=&\dsp (n-1)\det_\BB \left(A_1,-A_2,\cdots,- A_n\right)=(-1)^{n-1}(n-1)\det(A) \end{array}$$

(Déterminant d'une matrice compagnon) Correction

Calculer le déterminant d'ordre $n+1$ : $$\left| \begin{array}{lccccc} a_0 & -1 & 0 & \ldots & 0 & 0 \\ a_1 & x & -1 & \ldots & 0 & 0 \\ a_2 & 0 & x & -1 & \ldots & 0 \\ \vdots & \vdots & \vdots & \ddots & \vdots & \vdots \\ a_{n-1} & 0 & 0 & \ldots & x & -1 \\ a_n & 0 & 0 & \ldots & 0 & x \end{array} \right|$$

Correction

Notons $$\Delta_{n+1}(a_0,\cdots,a_n)= \left| \begin{array}{lccccc} a_0 & -1 & 0 & \ldots & 0 & 0 \\ a_1 & x & -1 & \ldots & 0 & 0 \\ a_2 & 0 & x & -1 & \ldots & 0 \\ \vdots & \vdots & \vdots & \ddots & \vdots & \vdots \\ a_{n-1} & 0 & 0 & \ldots & x & -1 \\ a_n & 0 & 0 & \ldots & 0 & x \end{array} \right|.$$ 1 ère méthode:
On développe selon la 1ère colonne: $$\Delta_{n+1}(a_0,\cdots,a_n)=a_0 \left| \begin{array}{ccccc} x & -1 & 0 & \ldots &0 \\ 0 & x & -1 & \ldots& \\ \vdots&0 & \ddots & \ddots& \\ & & \ddots & x & -1 \\ 0 & 0 & \ldots & 0 & x \end{array} \right|+\left| \begin{array}{lccccc} a_1 & -1 & \ldots & 0 & 0 \\ a_2 & x & -1 & \ldots & 0 \\ \vdots & \vdots & \ddots & \vdots & \vdots \\ a_{n-1} & 0 & \ldots & x & -1 \\ a_n & 0 & \ldots & 0 & x \end{array} \right|=a_0x^n+\Delta_n(a_1,\cdots,a_n)$$ puis par récurrence sur $n$, on trouve $$\boxed{\,\Delta_{n+1}(a_0,\cdots,a_n)=a_0x^n+a_1x^{n-1}+\cdots+a_{n-1}x+a_n\,}.$$ 2ère méthode:
On effectue l'opération $L_n \rightarrow L_n +xL_{n-1}+x^2L_{n-2}+\cdots x^{n-1}L_1+x^nL_0$, puis on développe le déterminant obtenu selon la dernière ligne.

($\star\star$) Correction

Calculer les déterminants suivants : $$ \begin{array}{lcl} \mathbf{a)\,}\, \left| \begin{array}{cccccc} x & a & a & \ldots & a & a \\ -a & x & a & \ldots & a & a \\ -a & -a & x & \ldots & a & a \\ \ldots & \ldots & \ldots & \ldots & \ldots & \ldots \\ -a & -a & \ldots & \ldots & -a & x \end{array} \right|,&& \mathbf{b)\,}\, \left| \begin{array}{cccccc} x & a_1 & a_2 & \ldots & a_{n-1} & 1 \\ a_1 & x & a_2 & \ldots & a_{n-1} & 1 \\ \ldots & \ldots & \ldots & \ldots & \ldots & \ldots \\ a_1 & a_2 & a_3 & \ldots &x & 1 \\ a_1 & a_2 & a_3 & \ldots & a_n & 1 \end {array} \right|\\ &&\\ \mathbf{c)\,}\,\left| \begin{array}{ccccc} -a_1 & a_1 &0 & \ldots & 0 \\ 0 & -a_2 & a_2 & \ldots &0 \\ \vdots & \vdots & \ddots & \vdots & \vdots \\ 0 & 0 & \ldots & -a_n & a_n \\ 1 & 1 & 1 & \ldots & 1 \end{array} \right|,&& \mathbf{d)\,}\,\left| \begin{array} {cccc} a_1+x & a_1 & \ldots & a_1 \\ a_2 & a_2+x & \ldots & a_2\\ \vdots & \vdots & \ddots & \vdots \\ a_n & a_n & \ldots & a_n + x \end{array} \right|,\\ &&\\ \mathbf{e)\,}\, \left| \begin{array}{ccccc} 1 & 2 & 3 & \ldots & n \\ n & 1 & 2 & \ldots & n-1 \\ n-1 & n & 1 & \ldots & n-2 \\ \vdots & \vdots & \vdots & \ddots & \vdots \\ 2 & 3 & 4 & \ldots & 1 \end{array} \right|,&&\mathbf{f)\,}\, \left| \begin{array} {ccccc} 1 & 2 & 3 & \ldots & n \\ x & 1 & 2 & \ldots & n-1 \\ x & x & 1 & \ldots & n-2 \\ \vdots & \vdots & \vdots & \ddots & \vdots \\ x & x & x & \ldots & 1 \end{array} \right|. \end{array} $$

Correction

$\mathbf{a)\,}\,$ Pour $y\in \K$, on définit $$f(y)=\left| \begin{array}{cccccc} x+y & a+y & a+y & \ldots & a+y & a+y \\ -a+y & x+y & a+y & \ldots & a+y & a+y \\ -a+y & -a+y & x+y & \ldots & a+y & a+y \\ \ldots & \ldots & \ldots & \ldots & \ldots & \ldots \\ -a+y & -a+y & \ldots & \ldots & -a+y & x+y \end{array} \right| ,$$ en utilisant les propriétés de déterminant on a $f(y)=\alpha y+\beta$. Il suffit de trouver $\alpha$ et $\beta$.
Or pour $y=a$, on a $f(a)=(x+a)^n$ et pour $y=-a$, on a $f(-a)=(x-a)^n$. On en déduit que $\beta=((x+a)^n+(x-a)^n \big)/2$.
Donc $\boxed{D=\dfrac{(x+a)^n+(x-a)^n}{2}} $
$\mathbf{b)\,}\,$ $ \prod{(x-a_i)} \quad$
$\mathbf{c)\,}\,$ $ (-1)^n(n+1) \prod{a_i} $
$\mathbf{d)\,}\,$ On soustrait de chaque colonne le précédent $C_j\longleftarrow C_j-C_{j-1}$ (pour $j\in \inter{2,n}$) ce qui donne, $$D= \left| \begin{array} {ccccc} 1 & 1 & 1 & \ldots & 1 \\ x & 1-x & 1 & \ldots & 1 \\ x & 0 & 1-x & \ddots & \vdots \\ \vdots & \vdots & \ddots & \ddots & 1 \\ x & 0 & \ldots &0& 1-x \end{array} \right| \overset{L_i\leftarrow L_i-L_{i+1}}{\leftrightsquigarrow} \left| \begin{array} {ccccc} 1-x & x & 0 & \ldots & 0 \\ 0 & 1-x & 0 & \ldots & 0 \\ \vdots & 0 & 1-x & \ddots & \vdots \\ 0 & \vdots & \ddots & \ddots & x \\ x & 0 & \ldots &0& 1-x \end{array} \right| $$ Enfin, on développe suivant la 1ère colonne, ce qui donne $\boxed{(1-x)^n-(-x)^n }\qquad$.
$\mathbf{e)\,}$ On ajoute à la première colonne les colonnes suivante puis on factorise par $n(n+1)/2$, on obtient: $$D=\frac{n(n+1)}{2}\left| \begin{array}{ccccc} 1 & 2 & 3 & \ldots & n \\ 1 & 1 & 2 & \ldots & n-1 \\ 1 & n & 1 & \ldots & n-2 \\ \vdots & \vdots & \vdots & \ddots & \vdots \\ 1 & 3 & 4 & \ldots & 1 \end{array} \right| $$ ensuite on soustrait de chaque colonne la colonne précédente $C_j\longleftarrow C_j-C_{j-1}$ (pour $j\in \inter{2,n}$) ce qui donne, $$D=\frac{n(n+1)}{2}\left| \begin{array}{ccccc} 1 & 1 & 1 & \ldots & 1 \\ 1 & 0 & 1 & \ldots & 1 \\ 1 & n-1 & 1-n & \ldots & 1 \\ \vdots & \vdots & \vdots & \ddots & \vdots \\ 1 & 2 & 1 & \ldots & 1-n \end{array} \right| $$ Enfin, on soustrait de chaque ligne (à partir de deuxième) la première ligne, $$ D=\frac{n(n+1)}{2}\left| \begin{array}{ccccc} 1 & 1 & 1 & \ldots & 1 \\ 0 & -1 & 0& \ldots & 0 \\ 0 & n-2 & -n & \ldots & 0 \\ \vdots & \vdots & \vdots & \ddots & \vdots \\ 0 & 1 & 0 & \ldots & -n \end{array} \right| =\boxed{\,\dfrac{(-n)^{n-1}(n+1)}{2}\,}.$$

($\star\star$) Correction

Calculer le déterminant d'ordre $2n$ suivant $\begin{vmatrix} a &0 &\cdots&\cdots& 0& b \\ 0 &\ddots& & & \diagup & 0 \\ \vdots& & a & b & &\vdots \\ \vdots& & b & a & &\vdots \\ 0 &\diagup& & & \ddots & 0 \\ b & 0 & \cdots & \cdots & 0 & a \end{vmatrix} $

Correction

On développe suivant la 1ère ligne, $$ D_n=a\begin{vmatrix} a & &&& & b&0 \\ &\ddots& & & \diagup & & \\ & & a & b & &&\vdots \\ & & b & a & &&\vdots \\ &\diagup& & & \ddots & & \\ b & & & & &\\ 0 & & & & & &a \end{vmatrix}+ (-1)^{2n+1}b\begin{vmatrix} 0&a & &\cdots&\cdots& & b \\ & &\ddots& & & \diagup & \\ &\vdots& & a & b & &\vdots \\ &\vdots& & b & a & &\vdots \\ & &\diagup& & & \ddots & 0 \\ & \diagup & & \cdots & \cdots & & a\\ b& & & & & & 0 \end{vmatrix}$$ On développe le premier selon la dernière ligne et le seconde suivant la première ligne, ce qui donne $$D_n=a^2D_{n-1}-b^2D_{n-1}\Longrightarrow D_n=(a^2-b^2)^n.$$

(ENSAM 2018) Correction

Soit $P\in \C_3[X]$, on définit la famille $\mathcal{F}=(P,XP,P',XP',X^2P')$ et on note $A$ la matrice de $\mathcal{F}$ dans la base canonique de $\C_4[X]$ et $D=\det(A)$.

  1. Montrer que $D=0$ ssi il existe $U\in \C_1[X]$ et $V\in \C_2[X]$ non nuls tels que $PU=P'V$.
  2. Montrer que $D=0$ ssi $P$ admet une racine multiplie.
  3. Calculer $D$ lorsque $P=aX^2+bX+c$.

Correction

  1. Si $P=0$ alors il n'y a rien à démontrer, supposons que $P\neq 0$.
    Si $D=0$ alors la famille $\mathcal{F}$ est liée, donc il existe $\alpha,\beta,\alpha',\beta',\gamma'\in \C$ non tous nuls tels que $$\alpha P+\beta XP+\alpha'P'+\beta' XP'+\gamma' X^2P'=0 \Leftrightarrow (\alpha+\beta X)P=-(\alpha'+\beta' X+\gamma' X^2)P'$$ Il suffit alors de poser $U=\alpha+\beta X\in \CC_1[X]$ et $V=-\alpha'-\beta'X-\gamma'X^2\in \C_2[X]$.
  2. Supposons que $P$ admet 3 racines distincts $a,b,c$ et que $D=0$, alors d'après la question précédente, on a $PU=P'V$ avec $\deg(U)\leq 1$ et $\deg (V)\leq 2$.
    Puisque $P(a)=P(b)=P(c)=0$ alors $V(a)=V(b)=V(c)=0$ ceci implique que $V=0$ donc $U=0$ ce qui est contredit le faite que $U,V$ non nuls!
  3. La matrice de la famille $\mathcal{F}$ dans la base canonique est $$M=\begin{pmatrix} c&0&b&0&0\\ b&c&2a&b&0\\ a&b&0&2a&b\\ 0&0&0&0&0 \end{pmatrix}$$ donc $D=0$.
    (ce résultat était prévisible puisque tous les polynômes de la famille $\mathcal{F}$ sont de degré $\leq 3$, donc $\mathcal{F}$ ne peut pas former une base de $\C_4[X]$).

(TPE-EIVP MP 2018) Correction

Calculer le déterminant de la matrice $A=\left(\sin(i+j)\right)_{i,j\in \inter{1,n}}$.

Correction

Si $n=2$ alors $$\det(A)=\begin{vmatrix} \sin(2)&\sin(3)\\ \sin(3)&\sin(4) \end{vmatrix}=\sin(2)\sin(4)-\sin(3)^2$$ Si $n>2$ alors $\det(A)=0$ car la matrice $A$ n'est pas inversible. En effet, la troisième colonne de la matrice est une combinaison linéaire de deux premières colonnes, puisque: $$\forall i\in \inter{1,n},\quad \sin(i+3)=2\cos(1)\sin(i+2)-\sin(i+1).$$

(CCP MP 2018) Correction

Soit $a\in \R$ et $A(a)=\begin{pmatrix} 0&-1&\cdots&-1\\ a&\ddots&\ddots&\vdots\\ \vdots&\ddots&\ddots&-1\\ a&\cdots&a&0 \end{pmatrix}$ et $U=((1))$, deux éléments de $\MM_n(\R)$.

  1. Calculer $\det (A(-1))$.
  2. On note $P(x)=\det(A(a)+xU)$. Montrer que $P$ est polynômial de degré inférieur ou égale à $1$.
  3. Calculer $P(-a)$ et $P(1)$, en déduire $\det(A(a))$.
  4. Étudier la continué de $a\mapsto \det(A(a))$ et retrouver la valeur de $\det(A(-1))$.

Correction

On ajoute à la première colonne les autres colonnes, on trouve $$ \det (A(-1))=-(n-1) \begin{vmatrix} 1&-1&\cdots&-1\\ 1&\ddots&\ddots&\vdots\\ \vdots&\ddots&\ddots&-1\\ 1&\cdots&-1&0 \end{vmatrix}$$

(Mines MP 2017) Correction

$a,b,c$ sont trois réels. On définit $A=\begin{pmatrix} a&b&\cdots&b\\ c&a&\ddots&\vdots\\ \vdots&\ddots&\ddots& b\\ c&\cdots&c&a \end{pmatrix}\in \MM_n(\R)$. On note $J$ la matrice carrée de taille $n$ dont tous les coefficients sont égaux à $1$.

  1. Déterminer le degré du polynôme $P(x)=\det(A+xJ)$
  2. En déduire $\det(A)$

Correction

  1. Soit $x\in \R$, on a $$P(x)=\begin{vmatrix} a+x&b+x&\cdots&b+x\\ c+x&a+x&\ddots&\vdots\\ \vdots&\ddots&\ddots& b+x\\ c+x&\cdots&c+x&a+x \end{vmatrix}$$ Ensuite on soustrait de chaque colonne (à partir de la deuxième colonne) la première colonne, on obtient $$P(x)=\begin{vmatrix} a+x&b-a&\cdots&b-a\\ c+x&a-c&\ddots&\vdots\\ \vdots&\ddots&\ddots& b-c\\ c+x&\cdots&0&a-c \end{vmatrix}$$ Ensuite, en développant suivant la première colonne, on trouve $P(x)=\alpha x+\beta$.
  2. Pour $x=-b$, on a $$P(-b)=-\alpha b+\beta= \begin{vmatrix} a-b&0&\cdots&0\\ c-b&a-b&\ddots&\vdots\\ \vdots&\ddots&\ddots& 0\\ c-b&\cdots&c+x&a-b \end{vmatrix}=(a-b)^n,$$ puis pour $x=-1$, on trouve également, $$P(-c)=-\alpha c+\beta =(a-c)^n$$ Si $b\neq c$, on trouve, $$P(x)=\dfrac{(a-b)^n-(a-c)^n}{c-b}x+\dfrac{c(a-b)^n-b(a-c)^n}{c-b},$$ en particulier, pour $x=0$, on a $\det(A)=\dfrac{c(a-b)^n-b(a-c)^n}{c-b}$.
    Si $c=b$, alors $\det (A)=a (a-b)^{n-1}$.

(Mines Ponts PC 2016) Correction

Soit $n\geq 2$. Trouver toutes les matrices $A\in \MM_n(\C)$ telles que: $$\forall X\in \MM_n(\C),\quad \det (X+A)=\det (X)+\det (A).$$

Correction

Soit $A\in \MM_n(\C)$ vérifiant la relation donnée, alors $$\det (A+A) = \det(2A)=2^n\det(A),\text{ et }\det(A+A)=\det(A)+\det(A)=2\det(A)$$ donc $\det(A)=0$.
Supposons que $A\neq 0$ alors il existe une colonne de $A$ non nulle, sans perdre de généralité, on peut supposer que $C_1$ la première colonne de $A$ est non nulle. On complète ensuite $C_1$ à une base de $\C^n$, Notons $\BB=(C_1,X_2,\cdots,X_n)$ cette base. On pose alors $X=(-C_1,X_2,\cdots,X_n)$, alors $$\det(X)=\det(-C_1, ,X_2,\cdots,X_n)=-\det(C_1,X_2,\cdots,X_n)\neq 0$$ et $\det (X+A)= \det(0,X_2+C_2,\cdots,C_n+X_n)=0\neq \det (X)$, ce qui est impossible!
Donc $A$ est la matrice nulle.

(Mines Ponts MP 2017) Correction

Montrer que, si $C\in \MM_n(\R)$ telle que pour tout $X\in \MM_n(\R),\,\det(C+X)=\det (X)$, alors $C$ est nulle.
Si $(A,B)\in \MM_n(\R)^2$ vérifie: $\forall X\in \MM_n(\R),\,\,\det(A+X)=\det (B+X)$, que dire de $A$ et $B$?

Correction

Supposons que $C\neq 0$, on note $C_j$ la colonne n° $j$ de la matrice $C$. Il existe $j_0\in \inter{1,n}$ tel que $C_{j_0}\neq 0$. On complète $C_{j_0}$ à une base $\BB$ de $\R^n$, avec $\BB=(X_1,\cdots,X_{j_0-1},C_{j_0},X_{j_0+1},\cdots,X_n)$.
On pose ensuite $X= (X_1,\cdots,X_{j_0-1},-C_{j_0},X_{j_0+1},\cdots,X_n)$ ce qui donne $$0 =\det(C_1+X_1,\cdots,C_{j_0-1}+X_{j_0-1},0,C_{j_0+1}+X_{j_0+1},\cdots,C_n+X_n)=\det (C+X)=\det(X)\neq 0$$ ce qui est impossible, donc $C=0$.
Pour la deuxième question, on prend $Y=X-B$, ce qui donne $$\forall Y\in \MM_n(\R),\, \det (A-B+Y)=\det(Y) \Longrightarrow A-B=0.$$

(ENSAM PSI 2016) Correction

Pour $x< y< z$, donner le signe de $\begin{vmatrix} 1&x&\ee^x\\ 1&y&\ee^y\\ 1&z&\ee^z \end{vmatrix}$.

Correction

On note $A=\begin{bmatrix} 1&x&\ee^x\\ 1&y&\ee^y\\ 1&z&\ee^z \end{bmatrix}$, alors $$\det (A)=\begin{vmatrix} 1&x&\ee^x\\ 0&y-x&\ee^y-\ee^x\\ 0&z-y&\ee^z-\ee^y \end{vmatrix}= (y-x)\left(\ee^z-\ee^y\right)-(z-y)\left(\ee^y-\ee^x\right)$$ Ensuite, $\left(\ee^z-\ee^y\right)\geq (z-y)\ee^y$ et $-\left(\ee^y-\ee^x\right)\geq -(y-x)\ee^y$, ce qui donne $\det (A)\geq 0$.

(E3A 2019) Correction

Soient $n \geqslant 3$ un entier naturel, $\gamma = (\gamma_0,\dots,\gamma_{n-1})$ une famille de $n$ éléments de $\mathbb{K} = \R$ ou $\C$, et: \[ V_\gamma = \begin{pmatrix}1 & 1 & \cdots & \cdots & 1 \\ \gamma_0 & \gamma_1 & \cdots & \cdots & \gamma_{n-1} \\ \gamma_0^2 & \gamma_1^2 & \cdots & \cdots & \gamma_{n-1}^2 \\ \vdots & \vdots & & & \vdots \\ \gamma_0^{n-1} & \gamma_1^{n-1} & \cdots & \cdots & \gamma_{n-1}^{n-1} \end{pmatrix} \in \MM_n(\mathbb{K}). \]

  1. Montrer que s'il existe un couple $(i,j) \in \inter{ 0,n-1}^2$ avec $i \neq j$ tel que $\gamma_i = \gamma_j$, alors: $\det(V_\gamma) = 0$.
  2. On suppose les $\gamma_i$ distincts deux à deux et on note $C_j$ la colonne d'indice $j$ de la matrice ${}^tV_\gamma$. Soient $\lambda_1,\lambda_2,\dots,\lambda_n$ des scalaires tels que: $\displaystyle\sum_{j=1}^n \lambda_jC_j = 0$. En utilisant le polynôme $P = \sum\limits_{j=1}^n \lambda_j X^{j-1}$, montrer: $\forall j \in \inter{ 1,n}$, $\lambda_j=0$. Que peut-on en déduire pour $\det(V_\gamma)$? On ne calculera pas $\det(V_\gamma)$.
  3. On suppose toujours que les $\gamma_k$ sont distincts deux à deux. Pour tout $k \in \inter{ 0,n-1}$, on définit sur $\R$ la fonction $\psi_k$ par: $x \mapsto \ee^{\gamma_k x}$.
    1. Soient $\left(m_k\right)_{k \in \inter{0,n-1}}$ un $n$-uplet de scalaires, et $\Psi = \displaystyle\sum_{k=0}^{n-1} m_k \psi_k$. Calculer les dérivées successives de $\Psi$ jusqu'à l'ordre $n-1$.
    2. En déduire que la famille $\left(\psi_k\right)_{k \in \inter{ 0,n-1}}$ est libre dans $\mathrm{C}^\infty(\R,\mathbb{K})$.
Correction

Correction du sujet disponible sur le site UPS

(DS 2018) Correction

Soient $(a_1,\cdots,a_n)\in \R^n$, on définit $$V_n(a_1,a_2\cdots,a_n)=\begin{vmatrix} 1&a_1&a_1^2&\cdots&\cdots&a_1^{n-1}\\ 1&a_2&a_2^2&\cdots&\cdots&a_2^{n-1}\\ \vdots&\vdots&\vdots & &&\vdots\\ 1&a_n&a_n^2&\cdots&\cdots&a_n^{n-1} \end{vmatrix}.$$

  1. Déterminer $V_2(a_1,a_2)$, $V_3(a_1,a_2,a_3)$.
  2. On définit pour $x\in \R$, $P(x)=V_n(a_1,\cdots,a_{n-1},x)$ i.e. $$P(x)= \begin{vmatrix} 1&a_1&a_1^2&\cdots&\cdots&a_1^{n-1}\\ 1&a_2&a_2^2&\cdots&\cdots&a_2^{n-1}\\ \vdots&\vdots& \vdots & &&\vdots\\ 1&a_{n-1}&a_{n-1}^2&\cdots&\cdots&a_{n-1}^{n-1}\\ 1&x&x^2&\cdots&x^{n-2}&x^{n-1} \end{vmatrix}.$$
    1. Rappeler la formule de développement de déterminant selon une ligne.
    2. En utilisant le développement par rapport à la $n$ère ligne, montrer que $P$ est une fonction polynômiale de degré $\leq n-1$.
    3. Que vaut le terme de $x^{n-1}$ dans $P(x)$?
    4. Pour $i\in \inter{1,n-1}$ calculer $P(a_i)$. En déduire les racines de $P$.
    5. En déduire que $P(x)=V_{n-1}(a_1,\cdots,a_{n-1})\dsp\prod_{i=1}^{n-1}(x-a_i)$
  3. Montrer alors que $V_n(a_1,\cdots,a_n)=V_{n-1}(a_1,\cdots,a_{n-1})\dsp\prod_{i=1}^{n-1}(a_n-a_i)$. Puis par récurrence sur $n\geq 2$, montrer $$V_n(a_1,\cdots,a_{n})=\dsp\prod_{1\leq i< j\leq n}(a_j-a_i)$$

Correction

Après le mardi de la deuxième d'après! Merci Lukas :)

($\star^{1/2}$) Correction

Soient $n\in \N^*$ impair, $A,B \in \MM_n(\R)$ telles que :~$ B {\,}^t (AB) + A = 0$. Montrer : $A \not\in\ GL_n(\R)$.

Correction

On a $B {\,}^t (AB) =-A$ ce qui donne $$\det(B {\,}^t (AB))=\det(-A)=(-1)^n\det(A)=-\det(A), \text{ or } \det(B {\,}^t (AB))=\det(B{\,}^t B{\,}^t A)=\det(B)^2\det(A)$$ on obtient alors $(1+\det(B)^2)\det(A)=0$ donc $\det(A)=0$ doit $A\not\in \mathrm{GL}_n(\R)$.

(CPP PC 2018) Correction

Soit $M\in \MM_4(\C),\,M=(C_1,\,C_2,\,C_3,\,C_4)$. On note $M'=(C_1+C_3,\,C_2+C_4,\,C_1-C_3,\,C_2-C_4)$.
Montrer que $\det (M')=4\det( M)$.

Correction

En utilisant les propriétés de la fonction $\det$, on trouve: $$\begin{array}{lcl} \det(M')&=& \det(C_1+C_3,\,C_2+C_4,\,C_1-C_3,\,C_2-C_4)\\ &=&\det (C_1,\,C_2+C_4,\,C_1-C_3,\,C_2-C_4)+\det (C_3,\,C_2+C_4,\,C_1-C_3,\,C_2-C_4)\\ &=&\det (C_1,\,C_2+C_4,\,C_1,\,C_2-C_4)-\det (C_1,\,C_2+C_4,\,C_3,\,C_2-C_4)\\ &&+\det (C_3,\,C_2+C_4,\,C_1,\,C_2-C_4)-\det (C_3,\,C_2+C_4,\,C_3,\,C_2-C_4)\\ &=&0-2\det (C_1,\,C_2+C_4,\,C_3,\,C_2-C_4)+0\\ &=&-2 \det (C_1,\,C_2,\,C_3,\,C_2-C_4)-2\det (C_1,\,C_4,\,C_3,\,C_2-C_4)\\ &=&2 \det (C_1,\,C_2,\,C_3,\,C_4)+2\det (C_1,\,C_2,\,C_3,\,C_4)=4\det(M) \end{array}$$

(CPP 2018) Correction

Soit $A= \begin{pmatrix} b &1 &\cdots&1\\ 1 &\ddots & \ddots &\vdots\\ \vdots& \ddots & \ddots &1\\ 1 & \cdots & 1 &b \end{pmatrix}\in \MM_n(\R)$, avec $b\in \R$. Déterminer $\det(A)$.

Correction

Il est clair que si $b=1$ alors $\det (A)=0$.
On effectue l'opération suivante, $C_1\longleftarrow C_1+\dsum_{k=2}^nC_k$, on obtient $$ \det (A)= \begin{vmatrix} b+n-1 &1 &\cdots&1\\ b+n-1 &\ddots & \ddots &\vdots\\ \vdots& \ddots & \ddots &1\\ b+n-1 & \cdots & 1 &b \end{vmatrix}= (b+n-1)\begin{vmatrix} 1 &1 &\cdots&1\\ 1 &b & \ddots &\vdots\\ \vdots& \ddots & \ddots &1\\ 1 & \cdots & 1 &b \end{vmatrix}= (b+n-1)\det (A')$$ Ensuite, dans la matrice $A'$, on effectue les opérations suivantes: $$\forall k\in \inter{2,n},\quad C_k\longleftarrow C_k-C_1$$ Ce qui donne $$\det(A)= (b+n-1) \begin{vmatrix} 1 &0 &\cdots&0\\ 1 &b-1 & \ddots &\vdots\\ \vdots& \ddots & \ddots &0\\ 1 & \cdots & 0 &b-1 \end{vmatrix}=(b+n-1)(b-1)^{n-1}$$

ENDO

($\star\star$) Correction

Soient $n \in \mathbb{N}^* $, $E$ un $\mathbb{K}$-espace vectoriel de dimension $n$, $f \in \LL(E)$ et $\BB = (e_1 ,...,e_n )$ une base de $E$. Montrer que pour tout $(x_1 ,...,x_n ) \in E^n$ : $$\sum\limits_{j = 1}^n {\det _\BB \left( {x_1 ,...,f(x_j ),...,x_n } \right)} = {\text{tr}}(f)\det _{\mathcal{B}} (x_1 ,...,x_n )$$

Correction

On distingue deux cas:

  1. Si la famille $(x_1,\cdots,x_n)$ est liée, on montre que $\sum\limits_{j = 1}^n {\det _\BB \left( {x_1 ,...,f(x_j ),...,x_n } \right)}=0$.
  2. Si la famille $(x_1,\cdots,x_n)$ est libre, alors elle forme une base de $E$, puis on écrit la matrice de $f$ dans cette base, et après calcul on trouve $$\sum\limits_{j = 1}^n {\det _\BB \left( {x_1 ,...,f(x_j ),...,x_n } \right)} = {\text{tr}}(f)\det _{\mathcal{B}} (x_1 ,...,x_n )$$

Dans les deux la relation est vraie.

($\star\star\star$) Correction

Soient $n\in \N^*$ et $f\in \LL(\R_n[X])$ définie par $f(P)=\dsp\sum_{k=0}^nP(k)(X-k)^k$. Calculer le déterminant de $f$.

Correction

Notons $e_k=(X-k)^k$ et $\BB =(e_k)_{0\leq k\leq n}$, il est clair que $\BB$ est une base de $\R_n[X]$. Pour calculer $\det(f)$ il suffit alors de calculer $\det(A)$ ou $A=(a_{i\,j})_{0\leq i,j\leq n}\in \MM_{n+1}(\R)$ la matrice de $f$ dans $\BB$. $$\forall j\in \inter{0,n},\quad f(e_j)=\sum_{i=0}^ne_j(i)e_i(X)=\sum_{i=0}^n(i-j)^je_i$$ donc $$a_{i\,j}=(i-j)^j =\sum_{k=0}^j{j\choose k} i^k(-j)^{j-k} =\sum_{k=0}^j \underset{n_{i\,k}}{\underbrace{i^k}} \underset{m_{k\,j}}{\underbrace{{j \choose k} (-j)^{j-k}}}.$$ On peut donc écrire $A$ comme le produit de deux matrice $N\times M$, avec $$N=(n_{i\,j}) \text{ où } n_{i\,j}=i^j,~~\text{ et } M=(m_{i\,j}) \text{ ou } m_{i\,j}=\left\{\begin{array}{lcl} {i\choose j}(-j)^{j-i}&\text{ si }&i\leq j\\ &&\\ 0&\text{ sinon}& \end{array}\right.$$ La matrice $M$ est une matrice triangulaire supérieur à éléments diagonaux égaux à $1$ donc $\det(M)=1$.
D'autre par, $N$ est la matrice de Vandermonde donc $$\det(N)=\prod_{0\leq i< j\leq n}(j-i)=\prod_{j=0}^n\left(\prod_{i=0}^j(j-i)\right)= \prod_{j=0}^n j!$$ On en déduit, $$\boxed{\det(f)=n!\times (n-1)!\cdots 2!\times 1=n (n-1)^2(n-2)^3\cdots 2^{n-1}}.$$

($\star\star$) Correction

Soient $A \in \MM_n (\mathbb{C})$ et $\varphi _A \in \LL(\MM (\mathbb{C}))$ déterminé par $\varphi _A (M) = AM$. Calculer la trace et le déterminant de $\varphi _A $.

Correction

On note, pour $j\in \inter{1,n}$ $V_j=\Vect (E_{i\,j},\,\,i\in \inter{1,n})$, i.e. $V_j$ est l'ensemble des matrice telle que la colonne $j$ est non nulle et les autres colonnes sont nulles (plus la matrice nulle).
Ensuite, on montre que $\varphi_A(V_j)\subset V_j$ et donc la matrice de $\varphi_A$ dans la base canonique de $\MM_n(\C)$ (avec une numérotation adaptée ) est une matrice diagonale par blocs.
Notons $\BB_j=(E_{1,j},E_{2,j},\cdots,E_{n\,j})$ la base de $V_j$, alors $$\forall i\in \inter{1,n},\,\, \varphi_A(E_{i,j})=AE_{i,j}=\dsum_{k=1}^n a_{k\,j} E_{k,j}$$ (En faite, ce produit donne la colonne n° $i$ de la matrice $A$ mais dans la position $j$)
On en déduit alors que la matrice de $(\varphi_A)_{\mid_{V_j}}=A$, donc la matrice de $\varphi_A$ dans la base $\BB=(\BB_1,\cdots,\BB_n)$ est $$M=\begin{pmatrix} A & 0 &\cdots&&0\\ 0 & A &0 &&\\ \vdots& \ddots &\ddots&\ddots&\\ & & & &0\\ 0 & & & 0 & A \end{pmatrix} \Longrightarrow \det(\varphi_A)=\det(A)^n. $$

(CCP PSI 2017) Correction

Soit $u\in \LL(\R^3)$. Étudier l'application $f$ définie par: $$f(x,y,z)=\det (u(x),y,z)+\det(x,u(y),z)+\det(x,y,u(z)).$$

Correction

Soit $x,y,z\in \R^3$, supposons que la famille $(x,y,z)$ est liée, on peut alors écrire par exemple $x=ay+bz$ avec $a,b\in \R$. Alors $$\begin{array}{lcl} f(x,y,z)&=&\det (u(x),y,z)+\det(x,u(y),z)+\det(x,y,u(z))\\ &=&\det (au(y)+bu(z),y,z)+\det(ay+bz,u(y),z)+\det(ay+bz,y,u(z))\\ &=&a\det (u(y),y,z)+b\det (u(z),y,z)\\ &&+a\det(y,u(y),z)+b\det(z,u(y),z)+a\det(y,y,u(z))+b\det(z,y,u(z))\\ &=&a\det (u(y),y,z)+b\det (u(z),y,z)+a\det(y,u(y),z)+b\det(z,y,u(z))=0 \end{array}$$ Supposons maintenant que la famille $(x,y,z)$ est libre, donc c'est une base de $\R^3$, notons $M=\begin{pmatrix} x_1&y_1&z_1\\ x_2&y_2&z_2\\ x_3&y_3&z_3 \end{pmatrix}$ la matrice de $u$ dans cette base, alors $$\begin{array}{lcl} f(x,y,z)&=&x_1\det (x,y,z)+x_2\det(y,y,z)+x_3\det(z,y,z)\\ &&+y_1\det (x,x,z)+y_2\det(x,y,z)+y_3\det(x,z,z)\\ &&+z_1\det(x,y,x)+z_2\det(y,y,z)+z_3\det(x,y,z)\\ &=&(x_1+y_2+z_3)\det (x,y,z)=\tr (u)\det(x,y,z). \end{array}$$ (trace de $u$ ne dépend pas de la base choisie). On en déduit alors que $f(x,y,z)=\tr (u)\det(x,y,z)$, ce résultat reste valable si la famille $(x,y,z)$ est liée.
Conclusion $\boxed{f(x,y,z)=\tr (u)\det (x,y,z)}$.

($\star$) Correction

Soient $A \in \MM_n (\mathbb{C})$ et $\varphi \in \LL(\MM (\mathbb{C}))$ déterminé par $\varphi (M) = {\,}^t M$. Calculer la trace et le déterminant de $\varphi$.

Correction

Soit $B_1$ une base de l'ensemble des matrice symétriques, et $B_2$ une base de l'ensemble des matrices antisymétriques. Alors $\BB=(B_1,\,B_2)$ est une base de $\MM_n(\C)$.
On a, $$\forall x\in B_1,\, \varphi(x)={\,}^t x=x,\, \forall y\in B_2,\, \varphi(y)={\,}^t y=-y,.$$ Donc la matrice de $\varphi$ dans la base $\BB$ est $\begin{pmatrix} I_{n_1}&0\\ 0&I_{n_2} \end{pmatrix}$ avec $n_1=\dfrac{n(n+1)}{2}$ et $n_2=\dfrac{n(n-1)}{2}$. On en déduit alors que $\tr (\varphi) n_1-n_2 =n$ et $\det (\varphi)=(-1)^{(n(n-1))/2}$.

($\star\star$) Correction

Soit $A\in \MM_n(\R)$ telle qu'il existe $\lambda>0$ avec $A^3=-\lambda A$. Montrer que le rang de $A$ est pair.

Correction

Notons $f$ l'application linéaire canoniquement associée à $A$, donc $f$ vérifie $f^3=-\lambda f$. On note $F=\im (f)\subset\R^n$. Notons d'abord que $F\cap \Ker(f)=\{0\}$, en effet si $x\in \im(f)\cap \Ker (f)$, on trouve: $$\exists z\in E,\, x=f(z) \text{ et } f(x)=0 =f^2(z) \text{ donc } 0=f^3(z)=-\lambda f(z)=-\lambda x \text{ soit } x=0.$$ Comme $F$ est stable par $f$, on définit alors $\fonct{v}{F}{F}{x}{f(x)}$. Vu que $F\cap \Ker(f)=\{0\}$, on en déduit que $\Ker v=\{0\}$, donc $v$ est bijective i.e. $\det (v)\neq 0$.
D'autre part, on a $$ \forall x\in F,\,\, v^3(x)=f^3(x)=-\lambda f(x)=-\lambda v(x) \longrightarrow \det(v)^3=(-1)^{\dim F}\lambda^{\dim F}\det (v)$$ donc $(-1)^{\dim F}>0$ autrement dit, $\dim F=2p,\, p\in \N$. donc $(-1)^{\dim F}>0$ autrement dit, $\dim F=2p,\, p\in \N$.

(Mines-Télécom PSI 2017) Correction

Soit $\fonct{\varphi}{\MM_n(\R)}{\MM_n(\R)}{M}{M+\tr (M)I_n}$. Déterminer $\det(\varphi)$ et $\tr (\varphi)$.
On pourra commencer par résoudre les systèmes $\varphi(M)=M$ et $\varphi(M)=(n+1)M$.

Correction

Soit $\BB'=(M_1,\cdots,M_{n^2-1})$ une base de $\Ker (tr)$ (on sait que la fonction $\tr$ est une forme linéaire non nulle, donc $\dim(\Ker(\tr))=\dim(\MM_n(\R))-1=n^2-1$).
Comme la matrice $I_n\not\in \Ker(\tr)$ alors $\BB=(\BB',I_n)$ est une base de $\MM_n(\R)$, de plus $$\forall x\in \BB',\, \, \varphi(x)=x+\tr(x)I_n=x,\quad \varphi(I_n)=I_n+nI_n=(n+1)I_n$$ donc la matrice de $\varphi$ dans la base $\BB$ est $\begin{pmatrix} I_{n^2-1}&0\\ 0&n+1 \end{pmatrix}$. On en déduit alors $$\boxed{\tr(\varphi)=n^2+n,\quad \det(\varphi)=n+1}.$$

BLOCS

($\star$) Correction

Soient $A,B,C,D \in \MM_n (\mathbb{K})$. On suppose que $D$ est inversible et que $C$ et $D$ commutent. Établir $$\det \left( { \begin{array}{cc} A & B \\ C & D \\ \end{array} } \right) = \det (AD - BC)$$

Correction

On a $$\begin{pmatrix} I_n&-BD^{-1}\\ 0&I_n \end{pmatrix} \begin{pmatrix} A&B\\C&D \end{pmatrix}=\begin{pmatrix} A-BD^{-1}C&0\\C&D \end{pmatrix}$$ ce qui donne $$\det\left( \begin{pmatrix} A&B\\C&D \end{pmatrix}\right)=\det\left(\begin{pmatrix} A-BD^{-1}C&0\\C&D \end{pmatrix}\right).$$ Puis, $$\begin{pmatrix} A-BD^{-1}C&0\\C&D \end{pmatrix}\begin{pmatrix} D&0\\ 0&D^{-1} \end{pmatrix}= \begin{pmatrix} AD-BD^{-1}CD&0\\ C&I_n \end{pmatrix}=\begin{pmatrix} AD-BC&0\\C&I_n \end{pmatrix}.$$ On en déduit $\det\left( \begin{pmatrix} A&B\\C&D \end{pmatrix}\right)=\det\left(\begin{pmatrix} A-BD^{-1}C&0\\C&D \end{pmatrix}\right)=\det\left(\begin{pmatrix} AD-BC&0\\C&I_n \end{pmatrix}\right)=\det(AD-BC)$.

($\star\star$) Correction

  1. Soient $A,B \in \MM_n (\R)$. Montrer que $\begin{vmatrix} A & B \\ - B & A \end{vmatrix} \geqslant 0$.
  2. Soient $A,B \in \MM_n (\R)$ telles que $AB = BA$. Montrer que $\det (A^2 + B^2 ) \geqslant 0$.
  3. Trouver un contre-exemple à 2 si $A$ et $B$ ne commutent pas.

Correction

  1. Pusique $A,\,B\in \MM_n(\R)\subset \MM_n(\C)$ alors $\overline{A}=A$ (idem pour $B$), on effectue alors les opérations suivantes: $$\begin{array}{lcl} \begin{vmatrix} A&B\\ -B&A \end{vmatrix}&=& (\ii)^n \begin{vmatrix} A&B\\ -\ii B&\ii A \end{vmatrix}=(\ii)^n (\ii)^n\begin{vmatrix} A&\ii B\\ -\ii B&\ii ^2A \end{vmatrix}= (-1)^n\begin{vmatrix} A&\ii B\\ -\ii B&- A \end{vmatrix}\\ &=& (-1)^n \begin{vmatrix} A&\ii B\\ A-\ii B&-A+\ii B \end{vmatrix} = (-1)^n \begin{vmatrix} A+\ii B&\ii B\\ 0&-A+\ii B \end{vmatrix}\\ &=&\det(A+\ii B)\det(A-\ii B)=\det(A+\ii B)\overline{\det(A+\ii B)}= \abs{\det(A+\ii B)}^2\geq 0. \end{array}$$
  2. On a $(A+\ii B)(A-\ii B)=A^2-\ii AB+\ii BA- \ii^2 B^2=A^2+B^2$ car $AB=BA$, donc $$\det(A^2+B^2)=\det(A+\ii B)\det(A-\ii B)=\det(A+\ii B)\overline{\det(A+\ii B)}=\abs{\det(A+\ii B)}^2\geq 0.$$
  3. Soient $A=\begin{pmatrix} 0&1&1\\ 1&0&0\\ 0&0&0 \end{pmatrix},\,B=\begin{pmatrix} 0&1&0\\ 0&0&0\\ 1&0&0 \end{pmatrix}$, on a $$A^2=\begin{pmatrix} 1&0&0\\ 0&1&1\\ 0&0&0 \end{pmatrix},\, B^2=\begin{pmatrix} 0&0&0\\ 0&0&0\\ 0&1&0 \end{pmatrix},\,AB=\begin{pmatrix} 1&0&0\\ 0&1&0\\ 0&0&0 \end{pmatrix},\, BA =\begin{pmatrix} 1&0&0\\ 0&0&0\\ 0&1&1 \end{pmatrix} $$ Donc $AB\neq BA$ et $\det(A^2+B^2)=-1$

($\star$) Correction

Soient $A,B\in \MM_n(\R)$. On pose $M=\begin{pmatrix} I_n&I_n&I_n\\ I_n&A&A^2\\ I_n&B&B^2 \end{pmatrix}\in \MM_{3n}(\R)$. Calculer $\det(M)$.

Correction

On a $$\begin{pmatrix} I_n&0&0\\ -I_n&I_n&0\\ -I_n&0&I_n \end{pmatrix}\begin{pmatrix} I_n&I_n&I_n\\ I_n&A&A^2\\ I_n&B&B^2 \end{pmatrix}=\begin{pmatrix} I_n&I_n&I_n\\ 0&A-I_n&A^2-I_n\\ 0&B-I_n&B^2-I_n \end{pmatrix}$$ On en déduit alors que $$\det\left(\begin{pmatrix} I_n&I_n&I_n\\ I_n&A&A^2\\ I_n&B&B^2 \end{pmatrix}\right)=\det\left(\begin{pmatrix} I_n&I_n&I_n\\ 0&A-I_n&A^2-I_n\\ 0&B-I_n&B^2-I_n \end{pmatrix}\right).$$ or $$\det\left(\begin{pmatrix} I_n&I_n&I_n\\ 0&A-I_n&A^2-I_n\\ 0&B-I_n&B^2-I_n \end{pmatrix}\right)=\det\left(\begin{pmatrix} A-I_n&A^2-I_n\\ B-I_n&B^2-I_n \end{pmatrix}\right)=\det\left(\begin{pmatrix} A-I_n&0\\ 0&B-I_n \end{pmatrix}\begin{pmatrix} I_n&A+I_n\\ I_n&B+I_n \end{pmatrix} \right)$$ D'autre part, on a $\begin{pmatrix} I_n&0\\ -I_n&I_n \end{pmatrix}\begin{pmatrix} I_n&A+I_n\\ I_n&B+I_n \end{pmatrix}=\begin{pmatrix} I_n&A+I_n\\ 0&B-A \end{pmatrix}$, ce qui donne finalement, $$\boxed{ \det\left(\begin{pmatrix} I_n&I_n&I_n\\ I_n&A&A^2\\ I_n&B&B^2 \end{pmatrix}\right)=\det(A-I_n)\det(B-I_n)\det(B-A)}.$$

($\star$) Correction

Soient $B \in \MM_n (\mathbb{R})$ et $A = \left( { \begin{array}{cc} {I_n } & B \\ B & {I_n } \\ \end{array} } \right) \in \MM_{2n} (\mathbb{R})$. A quelle condition la matrice $A$ est-elle inversible~? Donner son inverse quand cela est possible.

Correction

On a $\begin{pmatrix} I_n&0\\ B&-I_n \end{pmatrix}\begin{pmatrix} I_n&B\\ B&I_n \end{pmatrix}=\begin{pmatrix} I_n&B\\ 0&B^2-I_n \end{pmatrix}$, donc $\det\left(\begin{pmatrix} I_n&B\\B&I_n \end{pmatrix}\right)= \det(I_n-B^2)$ donc $$\begin{pmatrix} I_n&B\\B&I_n \end{pmatrix}\text{ inversible ssi } I_n-B^2 \text{ est inversible}.$$ Supposons que $I_n-B^2$ inversible, et notons $A^{-1}=\begin{pmatrix} X&Y\\Z&W \end{pmatrix}$ son inverse (avec $X,Y,Z,W\in \MM_n(\R)$, alors $$A\,A^{-1}=\begin{pmatrix} X+BZ&Y+BW\\ BX+Z&BY+W \end{pmatrix}=\begin{pmatrix} I_n&0\\ 0&I_n \end{pmatrix}\Longrightarrow \left\{\begin{array}{lcl} X+BZ&=&I_n\\ BX+Z&=&0\\ Y+BW&=&0\\ BY+W&=&I_n \end{array} \right.$$ Ce qui nous donne $$ \left\{\begin{array}{lcl} X-B^2X&=&I_n\\ Z&=&-BX\\ Y&=&-BW\\ -B^2W+W&=&I_n \end{array} \right. \Longrightarrow \left\{\begin{array}{lcl} X&=&(I_n-B^2)^{-1}\\ Z&=&-B(I_n-B^2)^{-1}\\ Y&=&-B(I_n-B^2)^{-1}\\ W&=&(I_n-B^2)^{-1} \end{array} \right.$$ Soit $\boxed{A^{-1}=\begin{pmatrix} (I_n-B^2)^{-1}&-B(I_n-B^2)^{-1}\\ -B(I_n-B^2)^{-1}&(I_n-B^2)^{-1} \end{pmatrix}}$.

($\star$) Correction

Soient $A,B,C,D$ des matrices carrées d'ordre $n$, réelles et commutant deux à deux. On suppose de plus que $A\in \mathrm{GL}_n(\R)$.
Montrer que la matrice $M = \left( { \begin{array}{cc} A & B \\ C & D \\ \end{array} } \right)$ est inversible si, et seulement si, $AD - BC$ l'est.

Correction

Comme $A\in \mathrm{GL}_n(\R)$ alors la matrice $\begin{pmatrix} I_n&0\\ -C&A \end{pmatrix}$ est inversible, de plus, on a $$ \begin{pmatrix} I_n&0\\ -C&A \end{pmatrix} \begin{pmatrix} A & B \\ C & D \end{pmatrix} =\begin{pmatrix} A&B\\ -CA+AC&-CB+AD \end{pmatrix} =\begin{pmatrix} A&B\\ 0&AD-CB \end{pmatrix}$$ Ceci implique $\det \begin{pmatrix} A & B \\ C & D \end{pmatrix} \neq 0$ ssi $\det \begin{pmatrix} A&B\\ 0&AD-CB \end{pmatrix} =\det (A)\det (AD-BC)\neq 0$, autrement dit $\det(AD-BC)\neq 0$. Soit $$\begin{pmatrix} A&B\\ C&D \end{pmatrix} \text{ est inversible ssi } AD-BC \text{ est inversible}$$

($\star\star$) Correction

Soient $p,q\in \N,\,A\in \MM_{p\,q}(\R),$ et $B\in \MM_{q\,p}(\R)$. Montrer que, $$\forall x\in \R,\quad x^q\det(xI_p-AB)=x^p\det(xI_q-BA).$$

Correction

Soit $x\in \R^*$, on a: $$\begin{pmatrix} xI_p&A\\B&I_q \end{pmatrix} \begin{pmatrix} I_p&0\\-B&xI_q \end{pmatrix} =\begin{pmatrix} xI_p-AB&xA\\0&xI_q \end{pmatrix},\quad \begin{pmatrix} xI_p&A\\B&I_q \end{pmatrix} \begin{pmatrix} I_p&-A\\0&xI_q \end{pmatrix} =\begin{pmatrix} xI_p&0\\B&xI_q-BA \end{pmatrix}.$$ Donc en passant par le déterminant, on obtient $$x^q\det\left(\begin{pmatrix} xI_p&A\\B&I_q \end{pmatrix}\right)=x^q\det(xI_p-AB),\quad x^q\det\left(\begin{pmatrix} xI_p&A\\B&I_q \end{pmatrix}\right)=x^p\det(xI_q-BA).$$ On en déduit alors $x^q\det(xI_p-AB)=x^p\det(xI_q-BA)$, cette relation reste vraie pour $x=0$.